Relay-Version: version B 2.10 5/3/83; site utzoo.UUCP Posting-Version: version B 2.10.2 9/18/84; site sdcsma.UUCP Path: utzoo!watmath!clyde!cbosgd!ihnp4!houxm!vax135!cornell!uw-beaver!tektronix!hplabs!sdcrdcf!sdcsma!ian From: ian@sdcsma.UUCP (Ian Ferris) Newsgroups: net.math Subject: Re: Need proof for density problem Message-ID: <127@sdcsma.UUCP> Date: Fri, 27-Sep-85 13:17:41 EDT Article-I.D.: sdcsma.127 Posted: Fri Sep 27 13:17:41 1985 Date-Received: Wed, 2-Oct-85 08:52:01 EDT References: <58@unc.unc.UUCP> Reply-To: ian@sdcsma.UUCP (Ian Ferris) Organization: System Development Corp. R+D, Santa Monica Lines: 26 In article <58@unc.unc.UUCP> southard@unc.UUCP (Scott Southard) writes: > >Is the set of numbers of the form 2^m * 3^n (that's 2 to the m power times >3 to the n power) where m and n are integers, dense in the positive >rational numbers? > If I'm not mistaken, if p and q are relatively prime integers, then the set S of numbers of the form p^m * q^n is dense in the positive reals. I suspect, since school has just started in most places, that this is a homework problem in either a junior level analysis course or an honors freshman calculus course. (If my suspicion is unfounded, I apologize). Since I think it's a bad idea to do other people's homework, I will only remark that one possible proof of this fact (the only one that I came up with) can be gotten by considering the logarithms of the numbers in S and proving that they are dense in the reals. This approach also requires you to know the following theorem: if r is any irrational number and e is any positive real number, then there exist integers m and n such that 0 < abs( m + n * r ) < e. However, thanks for the problem! I had a very pleasant noontime walk yesterday while working on it. Brought to you by Super Global Mega Corp .com